[Matematica pura, fisica, chimica, ecc.: problemi di allenamento del cervello non legati in alcun modo al commercio - pagina 450

 

In realtà c'è un'osservazione più generale (si può vedere dal tabulato MD): probabilmente tutte le scelte ragionevoli sono limitate a coppie di numeri 2^n e p (primo). Non l'ho provato, lo sto solo supponendo.

Ora, partendo da questo presupposto, facciamo qualcosa di concreto. La cosa più difficile nel dialogo dei saggi è l'ultima riga. È quello che finora richiede che si considerino molte opzioni. Supponiamo che abbiamo già avuto tre repliche e solo l'ultima rimane. Quante somme di MDS possono essere rappresentate come 2^n + prime?

Perché questa particolare decomposizione? Semplicemente perché B nell'ultima riga, considerando le possibili decomposizioni delle somme (vedi il mio post precedente) e i prodotti corrispondenti, avendo incontrato il prodotto 2*...*2*semplice, sa già in anticipo che solo una delle somme per esso può essere ammissibile, poiché solo una è dispari - se i numeri sono uguali a potenze di due e primi dispari. Questo dà immediatamente un vero candidato.

Quindi, andiamo.

11 = 2^2+7 = 2^3+3. Ci sono due candidati. Una seccatura in una volta sola.

17 = 2^2+13. Non ci sono più invii di questo tipo. Buon candidato.

23 = 2^2+19 = 2^4+7. Che peccato.

27 = 2^2+23 = 2^3+19 = 2^4+11. Tutto il resto è una seccatura.

29 = 2^4+13. Presentazione da sola. Un altro candidato.

35 = 2^2+31 = 2^4+19 = 2^5+3. Che peccato.

37 = 2^3+29 = 2^5+5 . Che peccato.

41 = 2^2 +37. Presentazione singolare. Candidato.

47 = 2^2+43 = 2^4+31. Che peccato.

51 = 2^2+47 = 2^3+43 . Che peccato.

53 = 2^4+37. La sottomissione è singolare. Candidato.

Così tra tutti gli SMD ci rimangono solo 4 somme ammissibili - 17, 29, 41, 53.

____________________________________________________________

Con 17 abbiamo trattato: B, avendo 17, calcola i numeri in modo unico nella quarta replica.

Da continuare. Ci restano solo tre numeri da analizzare per finire il problema.

__________________________________

P.S. Facciamo tutti e 4 i numeri brevi ed eleganti dopo tutto. Supponiamo che tre linee siano già state dette, e che rimanga solo l'ultima mossa di Sage B. Iniziamo con quelle che non passeranno, per arrivare rapidamente a quella principale.

29 = 4+25. П (=2*2*5*5) = 2*50 = 4*25 = 5*20 = 10*10. Le somme sono 52, 29, 25, 20. Solo 29 dalla lista verde sono appropriati. Questa è una soluzione a una cifra, cioè il candidato (numeri 4 e 25). Tuttavia, un'altra cifra singola che abbiamo già è 16 e 13. Quindi B non dirà la sua battuta.

41 = 16+25. П (=2*2*2*2*5*5) = 2*200 = 4*100 = 5*80 = 8*50 = 10*40 = 16*25 = 20*20. Le somme sono 202, 104, 85, 58, 50, 41 , 40. L'unico ammissibile è il 41, cioè il candidato (numeri 16 e 25). Tuttavia, un'altra cifra singola che abbiamo già è 4 e 37. Quindi B non dirà la sua battuta.

53 = 13+40. П (=2*2*2*5*13) = 2*260 = 4*130 = 5*104 = 8*65 = 10*52 = 13*40 = 20*26. Le somme sono 262, 134, 109, 73, 62, 53 , 46. L'unica somma ammissibile è, ovviamente, 53 (i numeri originali sono 13 e 40).Tuttavia, un'altra singola cifra che abbiamo già è 16 e 37. Quindi B non dirà la sua battuta.

E infine, 17. Non ho ancora trovato una breve prova della validità della soluzione. Sto pensando. Compilerò la prova completa più tardi, in modo che sia in un unico post. Ma il problema - ora, ora - è completamente risolto.

 

Trovato l'errore. Si chiama sovra-ottimizzazione. :)

C'era un superamento incompleto in un punto, una condizione di fine ciclo non corretta. Corretto.

// vedere le linee 68-69.

// for(uint i=2;i<=sqrt(n);i++) // ERROR!!!
for(uint i=2;i<n/2;i++) // questo è corretto.

Ora i risultati sono sorprendenti.

La soluzione è unica (S=17; P=52; a=4; b=13) fino alla somma massima == 867

Con max sum == 868, ci sono due soluzioni.

Ecco la stampa.

2011.01.15 18:33:11 MetaSage (EURUSD,M1) //+---- Somma massima = 867 -------------------+
2011.01.15 18:33:10 MetaSage (EURUSD,M1) S=17; P=52; a=4; b=13
2011.01.15 18:33:10 MetaSage (EURUSD,M1) //+---- Importo massimo = 867 -------------------+
2011.01.15 18:33:10 MetaSage (EURUSD,M1) //============== START ========================
2011.01.15 18:32:59 MetaSage (EURUSD,M1) //+---- Max = 868 -------------------+
2011.01.15 18:32:59 MetaSage (EURUSD,M1) S=65; P=244; a=4; b=61
2011.01.15 18:32:59 MetaSage (EURUSD,M1) S=17; P=52; a=4; b=13
2011.01.15 18:32:59 MetaSage (EURUSD,M1) //+---- Max = 868 -------------------+
2011.01.15 18:32:59 MetaSage (EURUSD,M1) //============== START ========================

File:
 

Quindi, questo compito ha un potenziale enorme, non un misero centinaio. Ho trovato il testo:

А вот что говорит RockMover, который решал эту задачу на компьютере: Следующая пара - 4 и 61, она появляется, когда наибольшее допустимое число - 437. (Если я ничего не напутал). В диапазоне примерно до 800 появляется еще пара (32, 131), а пара (16, 73) - только когда диапазон больше 900.

Non ho controllato più precisamente a causa della lentezza della macchina di calcolo, e non potevo usare il supercomputer Cray I, perché in primo luogo avrei dovuto portare via la gente dal lavoro, e in secondo luogo è comunque il fine settimana.

MD, fallo arrivare a un paio di migliaia, eh?

 

Prossima frontiera 1503 (2 decisioni) / 1504 (3 decisioni)

2011.01.15 18:50:34 MetaSage (EURUSD,M1) //+---- Max = 1504 -------------------+
2011.01.15 18:50:34 MetaSage (EURUSD,M1) S=163; P=4192; a=32; b=131
2011.01.15 18:50:34 MetaSage (EURUSD,M1) S=65; P=244; a=4; b=61
2011.01.15 18:50:34 MetaSage (EURUSD,M1) S=17; P=52; a=4; b=13
2011.01.15 18:50:34 MetaSage (EURUSD,M1) //+---- Max = 1504 -------------------+
2011.01.15 18:50:34 MetaSage (EURUSD,M1) //============== START ========================
2011.01.15 18:50:10 MetaSage (EURUSD,M1) //+---- Importo massimo = 1503 -------------------+
2011.01.15 18:50:09 MetaSage (EURUSD,M1) S=65; P=244; a=4; b=61
2011.01.15 18:50:09 MetaSage (EURUSD,M1) S=17; P=52; a=4; b=13
2011.01.15 18:50:09 MetaSage (EURUSD,M1) //+---- Importo massimo = 1503 -------------------+

Alexei > "E infine 17. Non ho ancora trovato una breve prova della validità della soluzione. Credo".

Beh, non ce ne sarà uno breve qui, perché tutto il dialogo è corretto. Ha bisogno di un'analisi completa. "Bae..."

 
Mathemat:

Quindi, questo compito ha un potenziale enorme, non un misero centinaio. Ho trovato il testo:

MD, fallo arrivare a un paio di migliaia, eh?

Nessun problema. Ma potrei farlo da solo, c'è un copione. O non ce l'hai su mt5? :)
 

Un enorme grazie a ValS per aver fatto scivolare un così grande e antico... bojang.

Allo stesso tempo, propongo di dare al problema il titolo di più figo del ramo.

MD, OK, lo eseguirò io stesso. Non ancora :)

 

A duemila 4 soluzioni, ma non ho cercato il confine - il computer è lento, è noioso passare manualmente attraverso il confine.

2011.01.15 18:59:16 MetaSage (EURUSD,M1) //+---- Importo massimo = 2000 -------------------+
2011.01.15 18:59:14 MetaSage (EURUSD,M1) S=163; P=4192; a=32; b=131
2011.01.15 18:59:14 14 MetaSage (EURUSD,M1) S=89; P=1168; a=16; b=73
2011.01.15 18:59:14 14 MetaSage (EURUSD,M1) S=65; P=244; a=4; b=61
2011.01.15 18:59:14 14 MetaSage (EURUSD,M1) S=17; P=52; a=4; b=13
2011.01.15 18:59:14 MetaSage (EURUSD,M1) //+---- Max = 2000 -------------------+

Forse è lento all'inizio, a causa della tabella di decomposizione dei moltiplicatori troppo grande.

Ho una tabella di dimensioni SMax*(SMax-1) lì, per sicurezza. Vedrò se posso ridurlo a una dimensione più piccola. Ho bisogno di un lemma per il prodotto massimo... :))

 
Mathemat:

1. Un enorme grazie a ValS per aver fatto scivolare un così grande e antico... bojang.

2. Allo stesso tempo, propongo di dare a questo problema il titolo di più figo del ramo.

3. MD, OK, lo eseguirò io stesso. Non ancora :)

Sono d'accordo su tutti e tre i punti.
 

Mathemat:

Quindi, questo compito ha un potenziale enorme, non un misero centinaio. Ho trovato il testo:

Ed ecco cosa dice RockMover, che ha risolto questo problema al computer: La prossima coppia è 4 e 61, e appare quando il numero più grande possibile è 437. (Se non mi sbaglio). Un'altra coppia (32, 131) appare nell'intervallo fino a circa 800, e la coppia (16, 73) appare solo quando l'intervallo è maggiore di 900.

Non ho controllato più precisamente a causa della lentezza della macchina calcolatrice, e non potevo usare il supercomputer Cray I, perché in primo luogo avrei dovuto portare via la gente dal lavoro, e in secondo luogo è comunque il fine settimana.

Il tuo RockMover sta mentendo spudoratamente. Non c'è nessuna lettera del genere. Ho controllato sul mio Cray... ;)
 
MetaDriver: Ho una tabella di dimensioni SMax*(SMax-1) nel caso. Ora penserò, forse sarà meno corretto. Ho bisogno di un lemma per il prodotto massimo... :))

In generale, bisogna eliminare le restrizioni sull'importo. Tutto il ragionamento rimane essenzialmente lo stesso, solo più di questo.

A giudicare dal fatto che nella citazione l'uomo ha bisogno del Cray 1, il suo algoritmo era meno ottimizzato del tuo :)